Integration of trigonometric function

Click For Summary
The integral of sin²(x)/(1+cos²(x)) can be approached by applying double angle formulas, specifically converting the squared trigonometric functions into a rational function of cos(2x). Utilizing the tangent half-angle substitution can further simplify the integration process. It is noted that partial fractions are not typically effective for integrating trigonometric functions, as they are better suited for rational expressions. Instead, simplifying the expression with trigonometric identities and then employing u-substitution or integration by parts is recommended. This method enhances the likelihood of successfully solving the integral.
Hashiramasenju
Messages
36
Reaction score
0

Homework Statement


I have included the LaTex version of the problem.
\int \frac{sin^2 x}{1+cos^2 x} dx

Homework Equations


Simplifying fraction
Partial fractions

The Attempt at a Solution


I have uploaded my attempt at the solution.
WIN_20160109_14_54_21_Pro.jpg
WIN_20160109_14_53_38_Pro.jpg
 
Physics news on Phys.org
I don't usually read handwritten solutions, and I never read ones printed sideways. Regarding your integral, I would suggest suggest using the double angle cosine formulas for your squared trig functions. This will give you a rational function of ##\cos(2x)##. Then the tangent half angle substitution will help you out. See:

https://en.wikipedia.org/wiki/Tangent_half-angle_substitution
 
Partial fractions and trig functions usually don't go together. Partial fractions are typically used to simplify rational expressions of a single variable.

For integrating rational expressions of trig functions, a typical approach is to simplify using a trig identity of some sort first, and then try to use either u-substitution or integration by parts, if those techniques might be useful.
 
Question: A clock's minute hand has length 4 and its hour hand has length 3. What is the distance between the tips at the moment when it is increasing most rapidly?(Putnam Exam Question) Answer: Making assumption that both the hands moves at constant angular velocities, the answer is ## \sqrt{7} .## But don't you think this assumption is somewhat doubtful and wrong?

Similar threads

  • · Replies 8 ·
Replies
8
Views
2K
  • · Replies 15 ·
Replies
15
Views
2K
Replies
3
Views
2K
  • · Replies 11 ·
Replies
11
Views
2K
  • · Replies 11 ·
Replies
11
Views
2K
  • · Replies 5 ·
Replies
5
Views
2K
  • · Replies 19 ·
Replies
19
Views
2K
  • · Replies 4 ·
Replies
4
Views
2K
  • · Replies 8 ·
Replies
8
Views
2K
  • · Replies 9 ·
Replies
9
Views
2K